LSAT and Law School Admissions Forum

Get expert LSAT preparation and law school admissions advice from PowerScore Test Preparation.

User avatar
 Dave Killoran
PowerScore Staff
  • PowerScore Staff
  • Posts: 5853
  • Joined: Mar 25, 2011
|
#88145
Complete Question Explanation
(The complete setup for this game can be found here: lsat/viewtopic.php?f=167&p=88139#p88139)

The correct answer choice is (E).

Answer choice (A) can be eliminated because, as discussed during the analysis of the fourth rule, when J is still on board when the van reaches F, G must still be on board when the van reaches S.

Answer choice (B) can be eliminated because from the first rule L must be either the first or second stop.

Answer choice (C) can be eliminated because from the third rule J is on board longer than V.

Answer choice (D) can be eliminated because from the second rule R is still on board when the van reaches M.

Thus, answer choice (E) is proven correct by process of elimination.
 josuecarolina
  • Posts: 24
  • Joined: Jul 20, 2012
|
#22158
Can you explain #19?
 Nicholas Bruno
PowerScore Staff
  • PowerScore Staff
  • Posts: 62
  • Joined: Sep 27, 2011
|
#22159
First, I would recommend that you read a prior post about the setup lsat/viewtopic.php?f=167&t=8582.

19)
A is wrong because G is before S but J is not before F. violates Rule 3.
B is wrong because L must be one or two
C is wrong because V must come before J (which it does not)
D is wrong because R cannot come before M.
Which leaves E as the correct answer

Let me know if this helps you out!

Get the most out of your LSAT Prep Plus subscription.

Analyze and track your performance with our Testing and Analytics Package.